LSAT and Law School Admissions Forum

Get expert LSAT preparation and law school admissions advice from PowerScore Test Preparation.

 Administrator
PowerScore Staff
  • PowerScore Staff
  • Posts: 8916
  • Joined: Feb 02, 2011
|
#41318
Complete Question Explanation
(The complete setup for this game can be found here: lsat/viewtopic.php?t=15716)

The correct answer choice is (D)

If the two French films are shown on day 3 and day 5, then there is not a third French film available to be shown, and answer choice (A) can be eliminated. The Not Laws created during the game setup eliminate answer choice (C) (I cannot be shown first) and answer choice (E) (G cannot be shown sixth). Answer choice (B) is incorrect because if G is shown first, then from the combination of the second and third rules I must be shown second, and N must be shown third, but that creates a conflict because F has already been assigned to day 3. Thus, answer choice (D) is correct.

Get the most out of your LSAT Prep Plus subscription.

Analyze and track your performance with our Testing and Analytics Package.